1. Trang chủ
  2. » Giáo Dục - Đào Tạo

Một số dạng toán số học tổ hợp

60 9 0

Đang tải... (xem toàn văn)

Tài liệu hạn chế xem trước, để xem đầy đủ mời bạn chọn Tải xuống

THÔNG TIN TÀI LIỆU

Thông tin cơ bản

Tiêu đề Một Số Dạng Toán Số Học - Tổ Hợp
Tác giả Huỳnh Văn Phong
Người hướng dẫn TS. Trịnh Đào Chiến
Trường học Trường Đại Học Quy Nhơn
Chuyên ngành Phương Pháp Toán Sơ Cấp
Thể loại Luận Văn Thạc Sĩ Toán Học
Năm xuất bản 2020
Thành phố Bình Định
Định dạng
Số trang 60
Dung lượng 311,76 KB

Các công cụ chuyển đổi và chỉnh sửa cho tài liệu này

Cấu trúc

  • M ÐU

  • MT S DANG TOÁN LIÊN QUAN ÐN H NHI PHÂN

    • Dang toán v h nhi phân

    • Dang toán quy v h nhi phân

  • MT S DANG TOÁN LIÊN QUAN ÐN BT BIN VÀ ÐN BIN

    • Bt bin và n bin bc nht, bc hai

    • Bt bin và n bin cua các phn t cc han

  • MT S DANG TOÁN LIÊN QUAN ÐN PHÂN HOACH TP HP

    • Dang toán yêu cu nêu phân hoach tp hp

    • Dang toán giai bng phng pháp phân hoach tp hp

    • Dang toán n yêu cu phân hoach tp hp

  • KT LUN

  • TÀI LIU THAM KHAO

Nội dung

Dạng toán về hệ nhị phân

Nội dung mục này tham khảo trong tài liệu [2]

Trong các bài toán tổ hợp, đặc biệt là bài toán đếm, thường xuất hiện những tình huống với hai khả năng xảy ra, như tô màu với hai màu xanh hoặc đỏ, di chuyển chỉ sang phải hoặc đi lên, hay phân loại số chẵn và số lẻ Những bài toán này có thể được biểu diễn dưới dạng các dãy nhị phân, bao gồm hai chữ số 0 và 1.

Bài toán 1.1.1 đề cập đến việc xếp hàng nhận xe đạp sau giờ học, với phí gửi xe là 1000 đồng Giả sử có k học sinh có tờ 1000 đồng và m học sinh có tờ 2000 đồng Dưới giả thiết rằng người giữ xe không có tiền thừa để trả, câu hỏi đặt ra là có bao nhiêu cách để các học sinh xếp hàng sao cho không em nào phải chờ nhận tiền thừa.

Bài toán này thuộc loại hai khả năng, trong đó mỗi học sinh có thể sở hữu tờ 1000 đồng hoặc tờ 2000 đồng Để hiểu rõ bản chất của bài toán, ta có thể lập tương ứng mỗi hàng học sinh với một dãy số gồm hai chữ số 0 và 1.

Giả sử mỗi học sinh có tờ 1000 đồng được biểu thị bằng chữ số 0, trong khi mỗi học sinh sở hữu tờ 2000 đồng sẽ được biểu thị bằng chữ số 1.

Mỗi hàng học sinh được biểu diễn bằng một dãy gồm k chữ số 0 và m chữ số 1 Để đảm bảo không có em nào phải chờ nhận tiền thừa, điều kiện cần thiết là k phải lớn hơn m.

Khi việc đếm số phần tử thỏa mãn điều kiện trở nên khó khăn, chúng ta có thể áp dụng phương pháp đếm phần bù, tức là đếm số phần tử không thỏa mãn điều kiện Đặc biệt, chúng ta cần xem xét số lượng hàng mà học sinh phải chờ đến lượt để nhận tiền thừa.

Theo cách tương ứng, một hàng xấu sẽ có sự tương ứng 1-1 với một dãy số gồm k chữ số 0 và m chữ số 1, trong đó tồn tại vị trí 2s + 1 có chữ số 1, và tại vị trí 2s, số chữ số 0 và số chữ số 1 là như nhau.

Nếu là người giữ xe thì ta gọi thêm một học sinh nào đó có tờ 1000 đồng lên đứng trước hàng xấu.

Phương pháp thực tiễn này cho phép chúng ta chuyển đổi mỗi hàng xấu với k chữ số 0 và m chữ số 1 thành một hàng mới có (k + 1) chữ số 0 và m chữ số 1, với chữ số 0 đứng đầu Đồng thời, trong (2s + 2) vị trí đầu tiên, số lượng chữ số 0 và chữ số 1 sẽ bằng nhau.

Khi thay đổi chữ số 0 thành 1 và chữ số 1 thành 0 ở vị trí đầu tiên của biểu thức (2s + 2), ta sẽ có một hàng mới với (k + 1) chữ số 0 và m chữ số 1, trong đó chữ số 1 sẽ đứng ở vị trí đầu tiên.

Lại bỏ đi chữ số1 đầu tiên này, ta được một hàng gồm (k+ 1) chữ số 0 và (m−1) chữ số 1.

Như vậy, mỗi hàng xấu gồm k chữ số 0, m chữ số 1 sẽ được tương ứng với một hàng gồm k + 1 chữ số 0 và m−1 chữ số 1 theo cách trên.

Ta chứng minh tương ứng như trên là một tương ứng 1-1.

Thật vây, xét một hàng tùy ý gồm (k+ 1) chữ số 0và (m−1) chữ số 1.

Ta thêm chữ số 1 vào đầu hàng sẽ được hàng mới gồm (k+ 1) chữ số 0 và m chữ số 1.

Do điều kiện k > m nên trong hàng này phải tồn tại vị trí (2s+ 2) mà từ đó trở lên, số chữ số 0 bằng số chữ số 1.

Chúng ta thực hiện việc thay đổi chữ số 0 thành chữ số 1 và ngược lại tại các vị trí từ đó trở đi, nhằm tạo ra một dãy số gồm k+1 chữ số 0 và m chữ số 1, với chữ số 0 ở đầu.

Bỏ chữ số 0 đầu tiên này ta nhậu được một hàng xấu.

Từ những tương ứng trên, ta suy ra số các hàng xấu gồm k chữ số

0, m chữ số 1 bàng số các hàng tùy ý gồm k + 1 chữ số 0, m−1 chữ số

Như vậy số cách xếp hàng sao cho không có học sinh nào phải chờ lấy tiền trả lại là

Bài toán 1.1.2 yêu cầu xếp 2k học sinh với chiều cao khác nhau thành hai hàng ngang Mục tiêu là sắp xếp sao cho chiều cao của học sinh trong mỗi hàng giảm dần, và học sinh đứng ở hàng trước phải cao hơn học sinh đứng ở hàng sau.

Hỏi có bao nhiêu cách xếp hàng thỏa mãn yêu cầu?

Phân tích Thoạt nhìn Bài toán 1.1.2 ta thấy bài toán khác hẳn với Bài toán 1.1.1.

Tuy nhiên, nếu ta đã nắm vững nguyên tắc hai khả năng thì nhận ra rằng cả hai bài toán trên thuộc cùng một loại.

Để giải quyết vấn đề phân loại học sinh, ta có thể xếp họ thành hai hàng Học sinh ở hàng trước sẽ cầm biển số 0, trong khi học sinh ở hàng sau sẽ cầm biển số 1.

Sau khi sắp xếp tất cả 2k em thành một hàng theo thứ tự chiều cao giảm dần, ta thu được một dãy gồm k chữ số 0 và k chữ số 1.

Để xếp hàng thỏa mãn điều kiện bài ra, mỗi vị trí (2s + 1) cần có số chữ số 0 từ vị trí đầu tiên đến đó lớn hơn hoặc bằng (s + 1) Áp dụng kết quả từ Bài toán 1.1.1, số cách học sinh xếp hàng thỏa mãn yêu cầu của Bài toán 1.1.2 được xác định như sau.

C 2k k −C 2k k+1 = (2k)! k!(k+ 1)!. Tiếp theo, luận văn trình bày chi tiết các kết quả được tham khảo từ tài liệu [1].

Dạng toán quy về hệ nhị phân

Peter có ba tài khoản ngân hàng với số tiền khác nhau và muốn thực hiện các giao dịch chuyển tiền giữa chúng Mục tiêu của anh là đảm bảo rằng số tiền trong tài khoản nhận tiền sẽ tăng gấp đôi sau mỗi lần chuyển.

Chứng minh rằng Peter luôn có thể chuyển tất cả tiền của mình vào hai tài khoản.

Peter có thể chuyển hết số tiền của mình vào một tài khoản được không?

Lời giải Từ yêu cầu của bài toán, ta cần phải xây dựng một thuật toán sao cho một tài khoản nào đó giảm ngặt dần (đơn biến).

Thuật toán được thể hiện ở các trường hợp sau đây.

Không mất tính tổng quát, giả sử số tiền trong ba tài khoản là

0< A ≤ B ≤ C Ta quy ước các kí hiệu sau:

+ Kí hiệu thứ nhất, chẳng hạn, (A, B, C) A

Số tiền trong ba tài khoản ban đầu là (A, B, C) Khi thực hiện chuyển khoản từ tài khoản thứ hai (có B đô-la) sang tài khoản thứ nhất (có A đô-la) một số tiền là A đô-la, số dư trong ba tài khoản sẽ trở thành (2A, B − A, C).

+ Kí hiệu thứ hai, chẳng hạn, 5 −−−→ cơ số 2 101 để chỉ số 5 trong hệ đếm thập phân khi đổi sang hệ đếm nhị phân sẽ thành số 101, nghĩa là

Ta xét 3 trường hợp sau đây.

- Trường hợp 1 Nếu 1.A ≤ B ≤2.A, thì ta thực hiện phép chuyển tiền như sau (A, B, C) A

Với phép chuyển này, số tiền trong tài khoản thứ hai sẽ giảm từ B xuống B −A.

Lưu ý, số 1 trong tích 1.A ở trên sẽ được đổi sang cơ số 2 như sau

1−−−→ cơ số 2 1, nghĩa là nghĩa là 1 = 1.2 0 Đẳng thức này tương ứng với phép chuyển A ←− A B (tương ứng với hệ số 1 của 2 0 ).

- Trường hợp 2 Nếu 2.A ≤ B ≤ 3.A, thì ta thực hiện liên tiếp hai phép chuyển tiền như sau (A, B, C) A

Với hai phép chuyển này, số tiền trong tài khoản thứ hai sẽ giảm từ B xuống B −2A.

Chú ý, số 2 trong tích 2.A ở trên sẽ được đổi sang cơ số 2 như sau

2−−−→ cơ số 2 10, nghĩa là nghĩa là 2 = 1.2 1 +0.2 0 Đẳng thức này tương ứng với phép chuyển liên tiếp A ←− A C (tương ứng với hệ số 0 của 2 0 ) và 2A ←− 2A B (tương ứng với hệ số 1 của 2 1 ).

- Trường hợp 3 Nếu 3.A ≤ B ≤4.A, thì ta thực hiện liên tiếp hai phép chuyển tiền như sau

Như vậy, với hai phép chuyển này, số tiền trong tài khoản thứ hai sẽ giảm từ B xuống B −3A.

Số 3 trong tích 3.A sẽ được chuyển sang cơ số 2, tương ứng với 3 = 1.2^1 + 1.2^0, hay 3 trong cơ số 2 là 11 Đẳng thức này phản ánh các phép chuyển liên tiếp A ←− A B và 2A ←− 2A B − A, với hệ số 1 của 2^0 và 2^1 Từ đây, có thể quy nạp để xây dựng một thuật toán nhằm giảm số tiền trong tài khoản thứ hai từ B xuống B − mA, với m là số nguyên dương sau một số phép chuyển tiền.

, luôn tồn tại duy nhất một số nguyên dương m sao cho mA ≤B ≤ (m + 1)A. Đổi số m sang cơ số 2, giả sử m −−−→ cơ số 2 mkm k−1 m1m0, nghĩa là m = m k 2 k +m k−1 2 k−1 + +m 1 2 1 +m 0 2 0 , với m k = 1, nghĩa là B = 2 k A+n, 0≤ n < A.

Trong quá trình chuyển tiền, chúng ta sẽ thực hiện từng lần chuyển theo thứ tự: lần đầu tiên tương ứng với giá trị m0, lần thứ hai với giá trị m1, và tiếp tục như vậy cho đến lần thứ k+1 tương ứng với giá trị mk, với i nằm trong khoảng từ 1 đến k+1.

+ nếum i−1 = 1, thì ta chuyển như sau: tài khoản 1 ←− A tài khoản 2.+ nếum i−1 = 0, thì ta chuyển như sau: tài khoản 1 ←− A tài khoản 3.

Vì B −2 k A = n, nên đến bước thứ k, tài khoản thứ nhất sẽ được

2 k A đô-la và tài khoản thứ hai sẽ còn B −2 k A= n đô-la.

Bây giờ, số tiền trong tài khoản thứ hai là nhỏ nhất Sắp xếp lại thứ tự số tiền trong ba tài khoản dưới dạng ban đầu

, và tiếp tục áp dụng thuật toán, ta lần lượt thu được dạng

Do đó, đến một bước nào đó, tài khoản thứ nhất sẽ bằng 0 Ta có điều phải chứng minh.

Peter có ba tài khoản ngân hàng với số dư lần lượt là 2018, 2019 và 2020 đô-la, minh họa cho thuật toán chuyển tiền.

- Bước 1 của thuật toán cho bộ 2018

A và 1 −−−→ cơ số 2 1, nghĩa là 1 = 1.2 0 , nên ta cần chuyển A← A −B.

! , kết thúc Thuật toán 1. Để chuẩn bị cho Bước 2 của thuật toán, ta sắp xếp lại bộ số trên theo thứ tự khởi đầu:

- Bước 2 của thuật toán cho bộ 1

A và 2020 −−−→ cơ số 2 11111100100, nghĩa là

.2 0 , nên ta cần chuyển theo thứ tự sau

! Để chuẩn bị cho Bước 3 của thuật toán, ta sắp xếp lại bộ số trên theo thứ tự khởi đầu:

- Bước 3 của thuật toán cho bộ 1024

Thực ra, đến đây, bước chuyển tiền tiếp theo đã quá rõ ràng! Tuy nhiên, ta thử thực hiện theo thuật toán.

A và 1 −−−→ cơ số 2 1, nghĩa là 1 = 1

.2 0 ,nên ta cần chuyển theo thứ tự sau

Thuật toán kết thúc khi sau một số bước chuyển, ba số tiền trong ba tài khoản ban đầu là (2018, 2019, 2020) đã được điều chỉnh thành (2048, 0, 4009).

Ta có điều phải chứng minh.

Trong trường hợp tổng quát, Peter không thể chuyển toàn bộ số tiền của mình vào một tài khoản duy nhất Ví dụ, nếu tổng số tiền của ba tài khoản ban đầu là một số lẻ, thì tổng số tiền của ba tài khoản cuối cùng sẽ trở thành một số chẵn, điều này là không hợp lý.

Bài toán 1.2.1 đã được giải quyết.

Bài toán 1.2.2 Giả sử f: N →N là hàm số thỏa mãn f (1) = 1, f (2n) = f (n) và f (2n+ 1) = f (2n) + 1; ∀n∈ N ∗

Tìm giá trị lớn nhất của f (n) với n ∈ {1; 2; 3; ; 2020}.

Phân tích Vì f (2n) được tính theo f (n) và f (2n+ 1) được tính theo f (2n).

Nên f (2n+ 1) cũng được tính theo f (n).

Do đó, để giải Bài toán 1.2.2 ta nghĩ đến việc viết các số tự nhiên n∈ [1; 2020] sang hệ nhị phân.

Lời giải Trước hết ta tính: f 102

Từ đây ta rút ra quy luật dưới dạng bổ đề sau:

Bổ đề Giá trị của hàm số f (n) bằng số chữ số 1 trong biểu diễn n dưới dạng hệ đếm nhị phân.

Chứng minh Giả sử Bổ đề đúng với mọi số số nguyên dương k < n Ta sẽ chứng minh Bổ đề đúng với n.

Thật vậy, xét hai trường hợp sau

- Trường hợp 1 Nếu n chẵn thì n= 2m = 10 2 m, với m ∈ N ∗

Trong hệ đếm nhị phân, để nhân một số với 2, ta chỉ cần thêm chữ số 0 vào bên phải số đó, do đó n = (10₂ m) và m có cùng chữ số 1 khi được biểu diễn trong hệ nhị phân.

Theo giả thiết quy nạp ta có f (m) bằng số chữ số 1 của m trong hệ nhị phân.

Theo đề bài, ta có f (n) = f (2m) = f (m).

Do đó f (n) đúng bằng số chữ số 1 của n trong hệ nhị phân.

- Trường hợp 2 Nếu nlẻ thìn = 2m+ 1 = 10 2 m+ 1, với m ∈ N ∗ Khi đó, trong hệ nhị phân, số chữ số 1 của n nhiều hơn số chữ số

1 của m là 1 (chính là chữ số 1 ở hàng đơn vị).

Theo đề bài, ta có: f (n) = f (2m+ 1) = f (m) + 1.

Suy ra f (n) bằng số chữ số 1 của m trong hệ nhị phân công thêm

1, tức là đúng bằng số chữ số 1 của n trong hệ nhị phân.

Bổ đề đã được chứng minh thông qua phương pháp quy nạp theo n Áp dụng Bổ đề này, bài toán 1.2.2 trở thành việc tìm số tự nhiên n không vượt quá 2020, với số lượng chữ số 1 nhiều nhất trong biểu diễn nhị phân của nó.

Ta có 11111111111 2 = 2 11 −1 = 2047 có 11 chữ số 1,

Mà 2020 < 2047, nên số n có nhiều nhất là 10 chữ số 1.

MỘT SỐ DẠNG TOÁN LIÊN

QUAN ĐẾN BẤT BIẾN VÀ ĐƠN BIẾN

Khái niệmbất biến và đơn biến đã được trình bày khá chi tiết trong tài liệu [1] và tài liệu [2].

Trong Chương 2, luận văn phân loại các bài toán Số học - Tổ hợp trong các kỳ thi chọn học sinh giỏi, nhấn mạnh việc ứng dụng bất biến và đơn biến trong quá trình giải quyết vấn đề.

Bất biến và đơn biến bậc nhất, bậc hai

Bài toán 2.1.1 Trên đường tròn cho sáu số theo thứ tự là1, 0, 1, 0, 0, 0. Mỗi bước biến đổi cho phép tăng hai số cạnh nhau lên cùng 1 đơn vị.

Hỏi có thể có sáu số bằng nhau sau một số hữu hạn bước biến đổi không?

Lời giải Ta xét giá trị biểu thức f (a 1 , a 2 , a 3 , a 4 , a 5 , a 6 ) =a 1 −a 2 +a 3 −a 4 +a 5 −a 6 , với a i , i= 1,6 là sáu số liên tiếp trên đường tròn.

Ta nhận thấy giá trị f (ai), với i = 1,6 không thay đổi sau mỗi bước biến đổi bất kì.

Ban đầu, giá trị của f là 2, nhưng nếu tất cả sáu số bằng nhau, giá trị của f sẽ trở thành 0 Điều này cho thấy sự mâu thuẫn, chứng tỏ rằng không thể có sáu số bằng nhau thông qua mọi phương pháp biến đổi.

Bình luận Việc nghĩ tới hàm f xác định như trên không phải ngẫu nhiên.

Với giả thiết, mỗi bước cho phép tăng hai số cạnh nhau lên cùng 1 đơn vị thì hiệu của hai số cạnh nhau luôn không đổi.

Ta gọi ai, với i = 1,6 như trên, biểu thức bất biến phải chứa các hiệu: a 1 −a 2 , a 2 −a 3 , a 3 −a 4 , a 4 −a 5 , a 5 −a 6 , a 6 −a 1

Từ đó dẫn tới hàm f có bất biến trong lời giải trên.

Bài toán 2.1.2 Xét dãy số vô hạn có sáu số ban đầu thứ tự là

Từ số hạng thứ bảy trong dãy số, mỗi số hạng được xác định bằng chữ số tận cùng của tổng sáu số hạng liền trước nó Câu hỏi đặt ra là liệu trong dãy vô hạn này có xuất hiện chuỗi 0, 1, 0, 1, 0, 1 hay không.

Lời giải Xét hàm tuyến tính g(n) = f(x n , x n+1 , x n+2 , x n+3 , x n+4 , x n+5 )

Theo điều kiện đề bài, dãy ban đầu được chuyển đổi thành dãy không âm {x_n} với n ∈ N, trong đó với mọi n ∈ N, ta có x_{n+6} ≡ (x_n + x_{n+1} + x_{n+2} + x_{n+3} + x_{n+4} + x_{n+5}) (mod 10) Từ đó, suy ra rằng g(n+1) - g(n) = 2 [x_{n+6} - (x_n + x_{n+1} + x_{n+2} + x_{n+3} + x_{n+4} + x_{n+5})] là một bội của 10 với mọi số tự nhiên n.

Mặt khác, vì f(1,0,1,0,1,0) = 8 và f(0,1,0,1,0,1) = 6 không đồng dư nhau theo modulo 10 nên trong dãy vô hạn đó không xuất hiện dãy thứ tự 0, 1, 0, 1, 0, 1.

Bình luận Tổng quát hóa Bài toán 2.1.2 khi ta thay 6 bởi i và điều kiện x n+i ≡ (x n + x n+1 +x n+2 + + x n+i−1 ) (mod (2i−2)), ∀i ∈ N.

Ta có hàm bất biến f (x n+1 , x n+2 , , x n+i ) ≡ (2x n + 4x n+1 + ã ã ã+ 2ix n+i−1 ) (mod(2i−2)).

Với điều kiện bất kì, chẳng hạn: x n+i ≡(a 0 x n + a 1 x n+1 + a 2 x n+2 + +a i−1 x n+i−1 ) (mod k), ∀i ∈ N, trong đó các hệ số a j (j = 0, i−1) là các số nguyên bất kì.

Ta tìm được bất biến thích hợp từ các dãy truy hồi tuyến tính.

+ Với dãy truy hồi cấp một dạngxn = px n−1 +q,trong đó p, q ∈ Z ta có x n −px n−1 = q là hằng số.

+ Với dãy truy hồi cấp hai dạng x n+1 = px n + qx n−1 , trong đó p, q ∈ Z, khi đó tồn tại các số u, v sao cho x n+1 −ux n = v(x n −ux n−1 ) = = v n+1 (x 1 −ux 0 ).

Chúng ta có thể áp dụng hệ thức tương tự cho các dãy truy hồi cấp cao hơn như cấp 3, cấp 4 hoặc cấp k bất kỳ Điều này bởi vì sai phân của một dãy truy hồi tuyến tính cấp k sẽ trở thành một dãy truy hồi tuyến tính cấp (k−1) với mọi k là số nguyên dương.

Với ví dụ trên, điều kiện là một dãy truy hồi cấp 7, nên tồn tại một bất biến bậc nhất như trên.

Hơn nữa, theo điều kiện bài toán là phải đồng dư theo modulo 10. Khi đó, ta tìm một biểu thức dạng g(n) = f(x n , x n+1 , x n+2 , x n+3 , x n+4 , x n+5 )

= a0xn +a1xn+1 +a2xn+2 +a3xn+3 +a4xn+4 +a5xn+5

Ta cần tìm ai, i= 0,5 để cho g(n) ≡g(n+ 1)(mod10), ∀n∈ N

Bài toán 2.1.3 Cho hai dãy số (x n ) n∈

Chứng minh rằng với mọi giá trị ban đầu x 0 , y 0 thì số giá trị nguyên phân biệt của hai dãy là hữu hạn.

Lời giải Để ý các hệ số trong công thức tổng quát lần lượt là

Suy ra x 2 n +y n 2 = = x 2 0 +y 0 2 := C là hằng số không âm, với mọi n ∈ N.

Như vậy mỗi dãy chỉ có hữu hạn giá trị nguyên phân biệt.

Bài toán 2.1.4 (IMO 1986) yêu cầu chứng minh rằng, khi đặt năm số nguyên dương trên một đường tròn, có thể thực hiện phép biến đổi cho ba số liên tiếp x, y, z (với y < 0) thành (x+y, -y, y+z) Phép biến đổi này sẽ dừng lại sau một số bước hữu hạn.

Lời giải Với các số x1, x2, x3, x4, x5 theo thứ tự trên đường tròn, xét hàm số f(x 1 , x 2 , x 3 , x 4 , x 5 ) : 5

Giả sử y = x 4 < 0, theo giả thiết, ta có hàm số f sau biến đổi là: f (x 1 , x 2 , x 3 +x 4 ,−x 4 , x 4 +x 5 ).

Khi đó hiệu giữa f sau và trước biến đổi là: f (x 1 , x 2 , x 3 +x 4 ,−x 4 , x 4 +x 5 )−f (x 1 , x 2 , x 3 , x 4 , x 5 )

Suy ra hàm f giảm thực sự sau mỗi bước biến đổi.

Hơn nữa, f chỉ nhận giá trị nguyên không âm.

Do đó, trong một khoảng chỉ có hữu hạn số nguyên phân biệt thì quá trình biến đổi phải dừng lại sau một số hữu hạn bước.

Bình luận Cần một đơn biến bậc hai, ta biến đổi

Ta nghĩ tới biểu thức (x i −x i±2 ).

Để giải quyết bài toán liên quan đến tính âm dương, việc kiểm soát tính chất của hàm f là rất quan trọng Do đó, chúng ta cần khảo sát một số hàm f cơ bản, trong đó hàm được đề cập là hàm đơn giản nhất đáp ứng cả ba yêu cầu đã nêu.

Bài toán 2.1.5 yêu cầu chứng minh rằng với n là một số nguyên dương lớn hơn 3 và n điểm trên một đường tròn, việc đổi chỗ b và c chỉ xảy ra hữu hạn lần nếu bốn số a, b, c, d theo thứ tự trên đường tròn thỏa mãn điều kiện (a−d)(b−c) < 0.

Lời giải Tương tự các ví dụ trên, ta xét hàm bậc hai sau f (x 1 , x 2 ,ã ã ã , x n ) =x 1 x 2 + x 2 x 3 + +x n x 1

Ta có thể kiểm tra f là hàm tăng thực sự sau mỗi phép biến đổi, và hơn nữa, f chỉ có hữu hạn giá trị.

Vậy quá trình biến đổi phải dừng lại sau một số bước hữu hạn.

Bài toán 2.1.6 Có 119 người trong 120 căn hộ Một căn hộ được gọi là quá tải nếu có nhiều hơn 15 thành viên.

Mỗi ngày, các thành viên trong những căn hộ quá tải gặp phải mâu thuẫn, dẫn đến việc họ phải chuyển sang các căn hộ khác nhau một cách ngẫu nhiên Liệu quá trình này có chắc chắn dừng lại hay không?

Lời giải Gọi ai là số thành viên của căn hộ thứ i, với i = 1,120. Xét biểu thức:

Giả sử căn hộ thứ j quá tải và gọi T là tập hợp những căn hộ khác mà các thành viên từ căn hộ a j chuyển sang.

Hiệu giữa biểu thứcS sau và trước lần chuyển căn hộ của các thành viên căn hộ thứ j là

Do đó S giảm thực sự sau mỗi lần chuyển nhà.

Hơn nữa S là một số nguyên và bị chặn dưới bởi 0 nên quá trình phải dừng lại sau hữu hạn bước.

Bình luận Trong lời giải trên ta vẫn cần một hàm đơn biến bậc hai sao cho các biến a j phải rời nhau.

Ta có thể chọn hàm f (ai) 120

Tuy nhiên, sau mỗi lần chuyển với căn hộ quá tải j thì hiệu

(ak + 1) 2 −a 2 j −X k∈T a 2 k có hệ số tự do dương.

Vì thế ta đánh giá vượt 120 sau một vài bước Để khử hệ số tự do, ta chọn biểu thức S như trên.

Bất biến và đơn biến của các phần tử cực hạn

Bài toán 2.2.1 (USAMO 2003) yêu cầu chứng minh rằng từ một hình lục giác với tổng các số nguyên không âm tại mỗi đỉnh bằng 2003, Bert có thể thực hiện các thao tác để tạo ra số 0 ở tất cả các đỉnh Cụ thể, Bert có thể chọn một đỉnh và thay thế số tại đó bằng giá trị tuyệt đối của hiệu hai số ở hai đỉnh lân cận Qua một chuỗi các thao tác như vậy, Bert sẽ có khả năng biến tất cả các số trên các đỉnh thành 0.

Để giải quyết bài toán, cần xây dựng một thuật toán nhằm đảm bảo rằng các số ở các đỉnh của hình lục giác sẽ giảm dần theo thứ tự.

Các thuật toán được thể hiện ở các trường hợp sau đây.

Lời giải Giả sử sáu số ban đầu làA, B, C, D, E, F, được viết theo thứ tự như hình vẽ.

Vì tổng của sáu số này bằng (2003)là số lẻ, nên các tổng: A+C+E hoặc

Không mất tính tổng quát, giả sử A+C +E là số lẻ và A ≥ C ≥ E ≥ 0.

Ta xét các trường hợp sau:

+Trường hợp 1 Nếu A, C, E > 0, thì ta sẽ xác định Thuật toán

I, gồm chuỗi các thao tác từ Bước 1 đến Bước 3, theo sơ đồ sau:

1" để chỉ A + C + E ở Bước 1 và "P i" để chỉ

A 0 +C 0 +E 0 ở Bước i, trong đó A 0 , C 0 , E 0 lần lượt là các vị trí tương ứng của A, C, E ở Bước 1.

Chú ý rằng, Thuật toán I sẽ chuyển các số A, C, E ở Bước 1 thành các số C − E, C, A − C không âm tương ứng ở Bước 3 Hơn nữa, vì

A + C + E > A + C − E nên tổng của chúng sẽ giảm ngặt, từ số lẻ

Thuật toán I đã tạo ra một đơn biến từ giá trị của đỉnh chứa số C trong các Bước 1, 2, 3 Đồng thời, nó cũng tạo ra một đơn biến từ tổng 3 số A, C, E ở Bước 1 đến tổng 3 số C−E, C, A−C ở Bước 3.

Do đó, nếu lặp lại Thuật toán I nhiều lần, một trong các số tương ứng với các vị trí của các số A, C, E ở Bước 1 sẽ bằng 0.

+Trường hợp 2 Nếu A ≥ C > 0 và E = 0, thì ta sẽ xác định Thuật toán II, gồm chuỗi các thao tác từ Bước 1 đến Bước 3, như sơ đồ sau:

Chú ý rằng, Thuật toán II sẽ chuyển các số A, C và E = 0 ở Bước

1 thành các số C, A−2C (hoặc 2C −A), E = 0 không âm tương ứng ở Bước 3.

Tổng của các số lẻ sẽ giảm từ A + C xuống 3C - A hoặc từ A + C xuống A - C Thuật toán II đã tạo ra một bất biến, đó là giá trị của đỉnh chứa số E = 0 trong các Bước 1, 2, 3, cùng với một đơn biến từ giá trị của.

A+C+E = A+C ở Bước 1 đến giá trị của C+ (2C −A) + 0 = 3C−A hoặc C + (A−2C) + 0 = A−C tương ứng ở Bước 3.

Nếu lặp lại Thuật toán II nhiều lần, sẽ có một thời điểm mà một trong các số tương ứng với vị trí của các số A và C ở Bước 1 trở thành 0.

+Trường hợp 3 Nếu A > 0 và C = E = 0, thì ta sẽ xác định Thuật toán III, gồm chuỗi các thao tác từ Bước 1 đến Bước 4 theo sơ đồ:

Thuật toán III chuyển đổi các số A, B, C = 0, D, E 0, F ở Bước 1 thành các số 0 tương ứng ở Bước 4, đồng thời tổng của chúng giảm từ số lẻ A xuống số 0 Qua quá trình này, thuật toán đã tạo ra một bất biến với giá trị của đỉnh chứa các số C = 0, E = 0 trong các Bước 1, 2, 3, 4, cùng với một đơn biến từ giá trị A + C + E = A ở Bước 1 đến giá trị 0 ở Bước 4.

Do đó, Thuật toán III là thuật toán cuối cùng để số 0 xuất hiện ở cả sáu đỉnh.

Như vậy, với các thuật toán trên, Bert có thể thực hiện một chuỗi các thao tác để cuối cùng số 0 sẽ xuất hiện ở sáu đỉnh.

Ta có điều phải chứng minh.

Sơ đồ dưới đây sẽ cho ta một minh họa của lời giải bài toán này.

Viên gạch hình lưỡi câu được định nghĩa là một khối hình gồm 6 ô vuông đơn vị, có thể được tạo ra bằng cách lật hoặc xoay viên gạch theo một góc nhất định Hình dạng này được minh họa trong bài toán 2.2.2 của IMO 2004.

Xác định tất cả các hình chữ nhật có kích thước m×n, với m và n là các số nguyên dương, có khả năng được lát kín bằng các viên gạch hình lưỡi câu Việc tìm kiếm này cần chú ý đến các quy tắc và điều kiện cần thiết để đảm bảo rằng các viên gạch có thể lắp ghép hoàn hảo mà không để lại khoảng trống.

Lời giải Với hai viên gạch hình lưỡi câu, chỉ có hai cách để ghép kín chúng với nhau như hình vẽ ở trên.

Ta gọi trường hợp thứ nhất là cặp ghép kín đủ (Hình 2a) và trường hợp thứ hai là cặp ghép kín thừa (Hình 2b).

Để tạo ra các cặp ghép kín có hình dạng m, n không thuộc tập {1,2,5}, kích thước hình chữ nhật m phải chia hết cho 12.

Với mãn = 12k, k ∈ N, ta xột hai trường hợp sau đậy.

- Trường hợp 1 Nếu m hoặc n chia hết cho 4, giả sử m chia hết cho 4, thì ta viết n dưới dạng n = 3l + 4r, với l, r ∈ N (cách phân tích này luôn tồn tại).

Khi đú, hỡnh chữ nhật kớch thước mãnluụn cú thể được lỏt kớn bởi k cặp ghép kín đủ.

Dưới đây là vài minh họa

+ Vỡ mãn= 24 = 2.12 = 3.8, nờn hỡnh chữ nhật kớch thước 3ì8 có thể được lát kín bởi hai cặp ghép kín đủ, ghép theo chiều rộng.

Hơn nữa, vỡ mãn= 24 = 2.12 = 4.6, nờn hỡnh chữ nhật kớch thước

4×6 có thể được lát kín bởi hai cặp ghép kín đủ, ghép theo chiều dài.

+ Vỡ mãn= 36 = 3.12 = 4.9, nờn hỡnh chữ nhật kớch thước 3ì12 có thể được lát kín bởi hai cặp ghép kín đủ, ghép theo chiều rộng.

Hơn nữa, vỡ mãn= 36 = 3.12 = 4.4, nờn hỡnh chữ nhật kớch thước

4×9 có thể được lát kín bởi hai cặp ghép kín đủ, ghép theo chiều dài.

- Trường hợp 2 Cả hai số m và n đều không chia hết cho 4. Suy ra m và n phải là số chẵn.

Ta chứng minh trường hợp này không thể lát được.

Thật vậy, giả sử ngược lại, hình chữ nhật kích thước m×n có thể lát được với m, n đều là số chẵn.

Chia hình chữ nhật thành m×n ô vuông và đánh số các hàng từ trên xuống dưới từ 1 đến m, đánh số các cột từ trái sang phải từ 1 đến n.

Ta gọi ô (p, q) là ô nằm ở giao của hàng thứ p và cột thứ q Với mỗi ô (p, q) tùy ý, ta sẽ điền số vào ô đó theo quy tắc sau.

+ Nếu chỉ một trong hai tọa độ p hoặc q chia hết cho 4 thì ta điền số 1 vào ô đó.

+ Nếu cả hai tọa độ p và q đều chia hết cho 4 thì ta điền số 2 vào ô đó.

+ Các trường hợp còn lại, ta điền số 0.

Với cách điền số như vậy, ta thu được một bất biến, đó là tổng các số trong hình 2a và tổng các số trong hình 2b đều là số lẻ.

Do đó, nếu hình chữ nhật được lát kín thì tổng các số trong toàn bộ hình chữ nhật này là số lẻ.

Tuy nhiên, vì m, n đều là số chẵn nên tổng các số trong toàn bộ hình chữ nhật này là số chẵn, mâu thuẫn.

Ta có điều phải chứng minh.

MỘT SỐ DẠNG TOÁN LIÊN

QUAN ĐẾN PHÂN HOẠCH TẬP HỢP

Lý thuyết phân hoạch tập hợp tuy đơn giản nhưng có ứng dụng phong phú trong giải quyết các bài toán khó Phương pháp này thường được sử dụng để giải nhanh và độc đáo các bài toán trong các kỳ thi chọn học sinh giỏi và Olympic Toán quốc tế.

Khái niệm phân hoạch tập hợp

Các tập hợp khác rỗng A 1 , A 2 , , A k được gọi là một phân hoạch của tập hợp A nếu

Mỗi tập con A i , với i = 1, k, được gọi là một thành phần của phân hoạch.

Chương 3 đề cập đến phương pháp phân hoạch tập hợp trong tài liệu [3].

Dạng toán yêu cầu nêu phân hoạch tập hợp

Bài toán 3.1.1 yêu cầu chứng minh rằng với một số hữu tỉ dương c khác 1, có thể chia tập hợp các số nguyên dương thành hai tập A và B khác nhau Điều kiện là cho mọi x, y thuộc cùng một tập A hoặc B, tích x y không bao giờ bằng c Việc phân hoạch này đảm bảo rằng các phần tử trong mỗi tập không tạo ra giá trị tích bằng c, từ đó tạo ra sự phân tách rõ ràng giữa hai tập hợp.

Lời giải Đặt c = p q, với p, q là hai số nguyên tố cùng nhau.

Khi số không không chia hết cho p và q, việc thêm chúng vào tập A hoặc tập B sẽ không làm thay đổi tính chất của hai tập hợp này và vẫn đảm bảo điều kiện của bài toán.

Xét các số chia hết chophoặc chia hết choq Chúng có dạngkp m q n , trong đó k không chia hết cho p và q.

Ta phân loại các số thành nhóm dựa trên bậc (m+n) của chúng Điều quan trọng là phải tách các số có cùng bậc vào những tập hợp khác nhau, vì nếu để chúng chung một tập hợp, có thể xảy ra trường hợp x y = c Để đáp ứng yêu cầu của bài toán, cần tránh trường hợp thương x y có kết quả mà trong đó p có bậc 1, vì khi đó có thể dẫn đến x y = p q = c.

Do đó, ta chỉ cần phân các số có cùng bậc p chẵn vào một tập hợp còn bậc p lẻ vào một tập hợp khác.

Ví dụ với bậc 5, nếu kp 0 q 5 , kp 2 q 3 , kp 4 q thuộc tập A thì kpq 4 , kp 3 q 2 , kp 5 q 0 thuộc tập hợp B với k không chia hết cho p, q.

Theo cách chia như trên ta phân hoạch được toàn bộ tập hợp các số nguyên dương thành hai tập hợp thỏa mãn điều kiện đề bài.

Bài toán 3.1.2 yêu cầu tìm số lượng phân hoạch của tập hợp A = {1, 2, 3, , n} thành ba tập hợp con A1, A2, A3, trong đó các tập hợp này có thể là tập rỗng Các điều kiện phân hoạch cần được thỏa mãn để đảm bảo tính chính xác của bài toán.

(i) Sau khi sắp xếp các phần tử của A 1 , A 2 , A 3 theo thứ tự tăng dần thì hai phần tử liên tiếp luôn có tính chẵn lẻ khác nhau;

(ii) Nếu cả ba tập A 1 , A 2 , A 3 đều không rỗng thì có đúng một tập có số nhỏ nhất là số chẵn.

Lời giải Ta có thể giả thiết thêm: (iii) 1 ∈ A1 và số nhỏ nhất trong A 2 bé hơn số nhỏ nhất trong A 3

Ta xây dựng các phân hoạch bằng cách xếp lần lượt các số1,2, , n vào A 1 , A 2 , A 3 theo các điều kiện (i),(ii),(iii).

Ta có 1 ∈ A 1 , còn số 2 có hai khả năng: 2 ∈ A 1 hoặc 2 ∈ A 2 (do điều kiện (iii)).

Nếu A 2 và A 3 còn là tập rỗng thì các số tiếp theo, do điều kiện (iii), đều chỉ có hai cách sắp xếp: vào A 1 hoặc A 2

Sau khi phần tử đầu tiên của A2 được sắp xếp, phần tử tiếp theo chỉ có hai lựa chọn: được xếp vào A2 hoặc A3, do tính chẵn lẻ của nó phải giống với phần tử cuối cùng của A1 tại thời điểm đó.

Khi A 3 còn là tập rỗng thì, do điều kiện (i), (ii), các số tiếp theo cũng chỉ có hai cách xếp vào hai trong ba tập A 1 , A 2 , A 3

Sau khi xếp phần tử đầu tiên của A3, có thể chứng minh bằng phương pháp quy nạp rằng mỗi số chỉ có thể được xếp vào hai trong ba tập A1, A2 và A3.

Nếu số k có thể được phân loại vào một trong hai tập Ai 1 hoặc Ai 2 và không thuộc tập Ai 3, thì (i1, i2, i3) là một hoán vị của (1,2,3) Điều này có nghĩa là k không có tính chẵn hay lẻ so với hai số cuối trong A i 1.

Ai 2 và cùng tính chẵn, lẻ với số cuối của Ai 3 lúc đó.

Giả sử xếp số k vào A i 1, thì ở bước tiếp theo, số k + 1 chỉ có thể được xếp vào A i 1 hoặc A i 3, theo điều kiện (i) Điều này cho thấy có hai khả năng sắp xếp cho số (k + 1).

Tóm lại, với mỗi số trong tập {1,2, , n}, trừ số 1, đều cho ta hai khả ngăng sắp xếp.

Vậy có 2 n−1 phân hoạch tập {1,2, , n}vào ba tập A1, A2, A3 theo yêu cầu bài toán.

Bài toán 3.1.3 yêu cầu chứng minh rằng từ tập hợp M gồm n số dương a1, a2, , an, ta có thể tạo ra tất cả các tập con T i khác rỗng và tính tổng các số trong mỗi tập hợp này, ký hiệu là s i Điều cần chứng minh là có thể phân chia tập hợp tất cả các số s i thành n tập hợp con khác rỗng, không giao nhau, sao cho tỉ số giữa hai số bất kỳ thuộc cùng một tập hợp con không vượt quá 2.

Lời giải Không mất tính tổng quát, giả sử a1 6 a2 6 6 an. Đặt S0 = 0, Sm = a1 +a2 + +am, với 1 6 m 6 n.

Kí hiệu P m = {s ∈ P\S m−1 < s 6 S m }, với m ∈ {1,2, , n} Ta chứng minh rằng cách chia P thành các tập P m như trên thỏa mãn điều kiện bài toán.

Muốn vậy ta chỉ cần chứng minh: Nếu b ∈ Pm thì 2b > Sm.

X k=1 a i k , nên tồn tại i k , với k = 1,2, , h sao cho i k > m.

Ta có điều phải chứng minh.

Dạng toán giải bằng phương pháp phân hoạch tập hợp

Bài toán 3.2.1 yêu cầu tìm số nguyên dương k nhỏ nhất trong tập hợp A gồm 16 số nguyên dương đầu tiên, sao cho trong mỗi tập con có k phần tử, luôn tồn tại hai số phân biệt a và b thỏa mãn điều kiện a² + b² là một số nguyên tố.

Giả sử k là số nguyên dương, trong mỗi tập con gồm k phần tử của tập A, luôn tồn tại hai số phân biệt a và b sao cho tổng bình phương của chúng, a² + b², là một số nguyên tố.

Xét tập con T gồm tất cả các số chẵn thuộc tập A Dễ thấy T có

8 phần tử và với a, b tùy ý thuộc T luôn có a 2 +b 2 là một hợp số từ đó suy ra k > 9.

Bằng cách tính toán và kiểm tra tất cả các tổng a² + b² với a, b thuộc tập A, chúng ta đã xác định được một phân hoạch gồm 8 tập hợp con, mỗi tập hợp chứa hai phần tử mà tổng bình phương của chúng là một số nguyên tố.

Theo nguyên lý Dirichlet, trong bất kỳ 9 phần tử nào của tập A, sẽ luôn có hai phần tử thuộc cùng một tập con trong phân hoạch Điều này có nghĩa là trong mỗi tập con gồm 9 phần tử của tập A, tồn tại hai số phân biệt a và b sao cho tổng bình phương của chúng, a² + b², là một số nguyên tố.

Từ các kết quả trên, ta suy ra min (k) = 9.

Bài toán 3.2.2 Cho p, q nguyên lẻ nguyên tố cùng nhau Chứng minh p−1

Ta phân tập A thành các tập hợp:

Vì |A| = |A 1 |+|A 2 | nên ta có điều phải chứng minh.

Đối với mỗi số tự nhiên n ∈ N, bài toán yêu cầu xác định số tự nhiên k ∈ N lớn nhất sao cho từ n phần tử, có thể chọn ra k tập con khác nhau Điều kiện là hai tập con bất kỳ trong số này phải có giao khác rỗng.

Ta cố định phần tử a1 và chỉ xét các tập con A1, A2, , Ak có chứa a 1 Khi đó, số tập con này bằng đúng số tập con của tập {a 2 , a 3 , , a n }.

Mà ∀i, j ∈ {1,2, , k} đều có a1 ∈ Ai∩Aj Nên k > 2 n−1

Giả sử chọn đượck > 2 n−1 tập con của tập X, mà giao của hai tập bất kì trong các tập được chọn ra đều khác rỗng.

Chia tất cả 2 n tập con của tập X thành 2 n−1 cặp sao cho trong mỗi cặp đều gồm tập con và phần bù của nó.

Vì số tập con chọn ra lớn hơn 2 n−1 nên theo nguyên lí Dirichlet, phải có ít nhất hai tập con đã chọn lập thành một cặp.

Do đó giao của hai tập này bằng rỗng, mâu thuẫn với tính chất của các tập hợp đã chọn ra.

Tóm lại, số tự nhiên k lớn nhất thõa mãn điều kiên bài toán là k = 2 n−1

, ta thu được đa thức f (x) = a 0 +a 1 x+a 2 x 2 + +a 100 x 100

Lời giải Trước hết ta chứng minh bổ đề sau.

Bổ đề Cho hai số tự nhiên n, k Xét tập hợp

Chứng minh Ta chứng minh bổ đề trên bằng phương pháp quy nạp theo n.

Khi n = 0 thì H 0,k = {k}, |H 0,k | = 1 = C k 0 : khẳng định đúng. Giả sử khẳng định đúng đến (n−1), với n > 1.

Xét phân hoạch H n,k = B 0 ∪B 1 ∪ ∪B k , trong đó (x 0 , x 1 , , x n ) ∈ B j nếu xn = j, với j ∈ {0,1,2, , k}.

Theo giả thiết quy nạp ta có: |B j | = |H n−1,k−j | = C n−1+k−j n−1 ,∀j 0,1,2, , k.

Dùng công thức C m i−1 + C m i = C m+1 i , ta có |H n,k | k

Do đó ta có điều phải chứng minh.

Lưu ý: |H n,k | = C n+k n −C n−1 n = C n+k n , với quy ước rằng C n−1 n = 0. Trở lại bài toán ở dạng tổng quát sau:

Vớim, n ∈ N, khai triểnf(x) = 1 +x+x 2 + +x m n+1 ta được đa thức f(x) =a 0 + a 1 x+a 2 x 2 + +a m(n+1) x m(n+1)

Viết cụ thể khai triển của f(x), ta có a i = |H n,i | = C n+i n , ∀i = 0,1,2, , m, |H n,i | > a i , nếu i > m + 1. Nói riêng: a 0 +a 1 +a 2 + +a m m

= C n+1+m n+1 Bài toán đã cho là trường hợp n+ 1 = m = 10.

Bài toán 3.2.5 đặt ra cho số nguyên n > 2 và định nghĩa S là tập hợp gồm n phần tử Các tập con A_i (với i = 1, m) là những tập con khác nhau, mỗi tập chứa ít nhất hai phần tử của S Điều kiện đưa ra là từ các quan hệ A_i ∩ A_j ≠ ∅, A_j ∩ A_k ≠ ∅, A_k ∩ A_i ≠ ∅, ta có thể suy ra rằng A_i ∩ A_j ∩ A_k ≠ ∅ Nhiệm vụ là chứng minh rằng số lượng m không vượt quá 2^(n-1) - 1.

Lời giải Ta chứng minh bằng phương pháp quy nạp theo n. Hiển nhiên, phát biểu ở đề bài đúng khi n = 2.

Giả sử n > 2 và phát biểu đúng với mọi số nguyên k < n.

Ta xét hai trường hợp:

Trường hợp 1: Không tồn tại i, j, với i = 1, m để cho A i ∪A j = S và |A i ∩A j | = 1.

Gọi x là phần tử tùy ý của S Số tất cả các tập Ai không chứa x lớn nhất bằng 2 n−2 −1, theo giả thiết quy nạp.

Số các tập con chứa x của S là 2 n−1

Nếux ∈ A i thì sẽ không tồn tại sốj nào để choA j = (S \A i )∪{x}, bằng không thì phải có |A i ∩A j | = 1.

Do vậy, quá lắm là một nửa các tập con chứa x của S xuất hiện dưới dạng các tập A i

Như thế, số lớn nhất các tập A i là 2 n−2 −1 + 2 n−2 = 2 n−1 −1. Trường hợp 2: Tồn tại một phần tử x ∈ S sao cho A 1 ∪A 2 = S và

Theo giả thiết quy nạp, số lớn nhất của các tậpA i sao cho A i ⊆ A 1 là 2 r −1.

Tương tự, số lớn nhất của các tập A i sao cho A i ⊆ A 2 là 2 s −1.

Nếu A i không phải là tập con của A 1 và A 2 thì A 1 ∩ A i 6= ∅,

A 2 ∩ A i 6= ∅ Vì A 1 ∩ A 2 6= ∅, nên A 1 ∩ A 2 ∩ A i 6= ∅ Như vậy ta có

Do đó, Ai = {x} ∪(Ai \A1)∪(Ai \A2).

Ngoài ra, do các tập khác rỗng (Ai \A1) và (Ai \A2) có thể được chọn tương ứng theo (2 s −1) và (2 r −1) cách, nên số lớn nhất các tập này là (2 s −1) (2 r −1).

Công thêm các kết kết quả riêng này vào, ta nhận được số lớn nhất các tập A i là 2 n−1 −1.

Bài toán 3.2.6 yêu cầu chứng minh rằng nếu định nghĩa f(r, n) là số phân hoạch của n dưới dạng b0 + b1 + b2 + + bs với điều kiện 0 ≤ i ≤ s−1 và bi > rbi+1, thì tồn tại một số nguyên dương n mà f(r, n) không bằng số phân hoạch của n thành các phần từ từ một tập hợp các số nguyên bất kỳ, ngoại trừ trường hợp r = 1.

Lời giải Ta chứng minh bài toán này bằng phản chứng.

Giả sử có một số r ≥ 2, tồn tại một tập hợp S r các số nguyên dương Nếu định nghĩa số phân hoạch của n thành các phần từ tập S r là g(r, n), thì ta có f(r, n) = g(r, n) cho mọi n.

Ta có f (r, 1) = 1 nên 1∈ S r Vì nếu không thì g(r, 1) = 0. Tiếp theo f (r, 2) = 1 nên 2∈/ S r vì nếu không thì g(r, 2) = 2.

Do f (r, 3) = f (r, 4) = = f (r, r+ 1) = 1 nên ta có được

Ta cóf (r, r+ 2) = 2do phân hoạch củar+2thỏa mãn b i > rb i+1 là r+ 2 và (r + 1) + 1.

Suy rar+ 2 ∈ S r vì nếu không thì g(r, r+ 2) = 1 Ta có bảng thể hiện các giá trị như sau: f (n), g(r, n) nếu n /∈ S r , g(r, n) nếu n ∈ S r

2r + 1 2 2 3 2r + 2 ∈/ S −r 2r + 2 3 2 3 2r + 3 ∈/ S −r 2r + 3 3 4 5 2r + 3 ∈/ S −r Tiếp theo ta xác định giá trị nhỏ nhất của n để f (r, n) = 3. Điều này xảy ra khi n = b 0 + b 1 + 1 với b 0 > rb 1 và b 1 > r hoặc n= b0 + 2 với b0 > 2r.

Giá trị n trong trường hợp thứ nhất nhỏ nhất là n = r 2 +r + 1

+ (r + 1) + 1 =r 2 + 2r + 3, trường hợp thứ hai là n = (2r + 1) + 2 = 2r + 3.

Do 2r + 3 < r 2 + 2r + 3, ∀r > 0 nên n = 2r + 3 là giá trị nhỏ nhất của n mà f (r, n) = 3 Trong trường hợp này, các phân hoạch của n là 2r + 3, (2r + 2) + 1, (2r + 1) + 2.

Do chỉ có 1, r + 2 ∈ S r nên dẫn đến có 2 phân hoạch của 2r + 3 thành các phần chỉ có (r + 2) + 1 + 1 + + 1, 1 + 1 + 1 + + 1.

Suy ra chỉ cần có 2r + 3∈ S r để có g(r, 2r + 3) = 3.

Tiếp theo ta thấy f (r, 2r + 4) = 3 do các phân hoạch của 2r + 4 mà b i > rb i+1 chỉ bao gồm 2r + 4, (2r + 3) + 1, (2r + 2) + 2, do r ≥ 2.

Mặt khác các phân hoạch của 2r+ 4 thành các phần trong tập hợp {1, r+ 2, 2r + 3} là (2r + 3) + 1, (r + 2) + (r + 2), (r + 2) + 1 + 1 + + 1, 1 + 1 + 1 + + 1.

Nên g(r, 2r + 3) = 4 nếu 2r + 4 ∈/ S r và g(r, 2r + 3) = 5 nếu 2r + 4 ∈ S r , mâu thuẫn.

Từ đây ta có điều cần chứng minh.

Bài toán 3.2.7 Gọi f (n), g(n) lần lượt là số phân hoạch của n thành số chẵn phần và số lẻ phần.

Chứng minh rằng f (n) −g(n) = (−1) r nếu n = 1 2 r(3r ±1), và f (n) =g(n) nếu ngược lại.

(số có dạng n = 1 2 r(3r ±1) được gọi là pentagonal number).

Lời giải Gọip e (n), p o (n)lần lượt là số phân hoạch của nthành số chẵn phần và số lẻ phần.

Ta sẽ thiết lập một tương ứng 1 – 1 giữa p e (n) và p o (n) trong trường hợp n6= 1 2 m(3m ±1).

Giả sử phân hoạch λ = (λ 1 , λ 2 , , λ r ) của n có thành phần nhỏ nhất là s(λ) = λr và σ(λ) là số các số nguyên dương liên tiếp giảm dần bắt đầu từ λ 1

Ta xét hai trường hợp sau

- Trường hợp 1 Nếu s(λ) ≤ σ(λ), ta thêm 1 vào mỗi phần trong s(λ) thành phần lớn nhất của phân hoạch λ và bỏ đi thành phần nhỏ nhất.

- Trường hợp 2 Nếu s(λ) > σ(λ), ta trừ 1 vào mỗi phần trong σ(λ) phần lớn nhất của phân hoạch λ và thêm một thành phần nhỏ nhất có kích thước σ(λ).

Trong hai trường hợp thay đổi tính chẵn lẻ của số phần trong phân hoạch, có thể nhận thấy sự tồn tại của một tương ứng một-một giữa p e (n) và p o (n).

Tuy nhiên, trường hợp 2 ở trên sẽ không còn đúng.

Do đó tùy theo tính chẵn lẻ của r mà có sự chênh lệch giữa p e (n) và po(n).

Bài toán 3.2.7 đã được chứng.

Bài toán 3.2.8 Gọi S(n) là số cách phân hoạch có tính thứ tự của n thành các số 1, 3, 4.

Chứng minh rằng S(2n) là số chính phương.

Giải Ta tính được công thức truy hồi

Tính lần lượt các số hạng của dãy, ta thấy

Do đó ta dự đoán và chứng minh S(2n) = F n+1 2 với F n là số Fibonacci thứ n.

Bài toán 3.2.9 Từ việc so sánh phân hoạch của n thành các số 1, 2.

Giải Ta sử dụng phương pháp đếm bằng 2 cách Trước hết, để tính số phân hoạch của n thành các số 1, 2 thì ta có công thức

Ta xét số nghiệm của phương trình x+ 2y = n.

Mỗi nghiệm sẽ có dạng

Mỗi hoán vị của các bộ số này tạo thành một phân hoạch thỏa mãn đề bài Ta tính được số hoán vị là

So sánh 2 kết quả ta có điều phải chứng minh.

Dạng toán ẩn yêu cầu phân hoạch tập hợp

Là phân hoạch mà đề toán ẩn yêu cầu đưa ra một phân hoạch thỏa mãn yêu cầu nào đó và ta phải làm việc này.

Bài toán 3.3.1 yêu cầu giải quyết một bảng ô vuông (a ij ) với các chỉ số i và j từ 1 đến n, điền các số từ 1 đến n² theo thứ tự từ trái sang phải và từ trên xuống dưới Sau đó, các hàng của bảng được ghép lại thành một dãy X, trong khi các cột được ghép thành một dãy Y Nhiệm vụ là xác định số phép biến đổi tối thiểu cần thiết để chuyển đổi dãy X thành dãy Y bằng cách hoán đổi vị trí của hai số bất kỳ.

Lời giải Chú ý rằng để biến đổi các số trong 2 chu trình khác nhau của một bộ k số, ta cần ít nhất k −1 cách Do đó, ta xây dựng dãy

X, Y trong trường hợp nhỏ và dự đoán số chu trình rời nhau, tính toán số phép biến đổi rồi tổng quát lên.

Bài toán 3.3.2 yêu cầu xác định tập A gồm tất cả các hoán vị a = (a1, a2, , a2003) của 2003 số nguyên dương đầu tiên, với điều kiện rằng không tồn tại tập con S nào của A thỏa mãn {ak | k ∈ S} = S.

1 Tìm giá trị nhỏ nhất của d(a), gọi giá trị nhỏ nhất đó là d 0

2 Tìm tất cả các hoán vị a ∈ A thỏa mãn d(a) =d 0

Lời giải Điều kiện đề bài nói lên rằng hoán vị đã cho có đúng 1 chu trình, với mỗi hoán vị bất kỳ thuộc A, ta xét ánh xạ f :i →a i

, k = 1, 2003 và m < 2003 là số tự nhiên thỏa mãn f m (1) = 2003.

Ta có đánh giá 2003 = |f m (1)−1| ≤ s 1 + s 2 + + s m + 1, và

Do d(a) là số chẵn nên d(a) ≥ 4.2003−4 = 8006.

Bài toán 3.3.3 Cho số nguyên dương k Dãy số (xn), n = 1, 2, 3, được xác định như sau:

(ii) Với mỗi số nguyên dương n ≥ 1 thì x n+1 là số nguyên dương bé nhất không thuộc tập hợp

Chứng minh rằng tồn tại số thực a sao cho x n = bnac, ∀n ∈ N ∗

Lời giải Xét đa thức f (x) =x 2 + (k−2) +k.

Gọi a là nghiệm dương của f (x).

Ta thấy a ∈ (1, 2) và a là số vô tỉ. Đặt b = a+ k thì 1 a + 1 b = 1.

Theo Định lý Beatty thì các giá trị của hàm số f (n) = [na], g(n) = [nb] +kn tại 1, 2, 3, tạo thành phân hoạch của N ∗

Bằng quy nạp ta chứng minh được xn = f (n).

Bài toán 3.3.4 yêu cầu chứng minh rằng tập hợp các số tự nhiên có thể được tô bằng hai màu với hai điều kiện: Thứ nhất, đối với mỗi số nguyên tố p và mỗi số tự nhiên n, các số p^n, p^(n+1) và p^(n+2) không được tô cùng màu Thứ hai, không tồn tại một cấp số nhân vô hạn của các số tự nhiên có cùng màu.

Lời giải Ta giả sử 2 màu cần tô là A, B.

Xét bốn nhóm màu AAB, ABA, BBA, BAB, ta xây dựng một dãy vô hạn S bằng cách sắp xếp các nhóm màu theo quy tắc: nhóm thứ nhất xuất hiện 1 lần, nhóm thứ hai 2 lần, nhóm thứ ba 3 lần, nhóm thứ tư 4 lần, sau đó lặp lại nhóm thứ nhất 5 lần, nhóm thứ hai 6 lần, và tiếp tục theo chu trình này.

Gọi S(k) là màu thứ k trong dãy, tính từ trái sang (chẳng hạn

S(2) = A, S(5) = B) và C(k) là màu tô cho số tự nhiên k, tất nhiên

Khi đó ta sẽ tô màu với quy tắc sau

Nếu số tự nhiên n = p a 1 1 p a 2 2 p a 3 3 p a k k thì

Chúng tôi sẽ chứng minh quy tắc này đáp ứng yêu cầu của đề bài Đầu tiên, điều kiện thứ nhất được rút ra từ việc ba màu liên tiếp trong S không thể đồng thời giống nhau Thứ hai, điều kiện này được chứng minh thông qua phương pháp phản chứng.

Trong một ngôi làng, có 5 người, mỗi người hoặc nói thật hoặc nói dối Khi một khách du lịch hỏi họ về số người nói thật trong hàng, mỗi người sẽ đưa ra một câu trả lời từ 0 đến 5 Câu hỏi đặt ra là có bao nhiêu tập hợp đa (multiset) các câu trả lời mà người khách có thể nhận được Việc phân tích các khả năng trả lời của 5 người này sẽ giúp xác định số lượng các multiset khả thi dựa trên quy tắc nói thật và nói dối của họ.

Lời giải Trước hết ta có bổ đề về multi-sets: Số các multi-sets có n phần tử lấy từ tập hợp gồm k phần tử là C n+k−1 k

Ta sẽ chọn các bộ có dạng (a, b, c, d, e) trong đó 0 ≤ a ≤ b ≤ c ≤ d ≤e ≤ 5 sao cho tồn tại một trường hợp trong thực tế thỏa mãn.

Ta xét 6 trường hợp sau:

1) Nếu a, b, c, d, e đều là các số dương thì rõ ràng luôn tồn tại trường hợp như thế nếu tất cả 5 người đều nói dối Số bộ như thế là

C 9 5 = 126 Nếu trong các sốa, b, c, d, e mà có một số bằng 0thì phải có

2) Nếu a = b= c = d = e= 0 thì không có trường hợp nào thỏa.

3) Nếu a = b = c = d = 0, e > 0 thì chỉ có bộ (0, 0, 0, 0, 1) thỏa với trường hợp 1 người nói thật, 4 người nói dối.

4) Nếu a = b = c = 0, d, e > 0 thì ta xét các trường hợp sau:+ Nếu có 1 người nói thật thì d = 1, e 6= 1, ta có 4 bộ như thế.

+ Nếu có 2 người nói thật thì d = e= 2, có 1 bộ.

Vì không thể có nhiều hơn 2 người nói thật nên có 5 bộ thỏa mãn đề bài.

+ Nếu có 1 người nói thật thì c = 1, d, e 6= 1 nên có tất cả

+ Nếu có 2 người nói thật thì 2 trong số c, d, e bằng 2 và số còn lại khác 2, có tất cả 4 bộ.

Do có 1 bộ trùng lại ở trên là (0, 0, 2, 2, 1) nên chỉ xét 3 bộ. + Nếu có 3 người nói thật thì c = d = e = 3 và có 1 bộ.

Do đó, có tất cả 10 + 3 + 1 = 14 bộ thỏa mãn.

+ Nếu có một người nói thật thì b = 1 và c, d, e ≥ 2, ta có

+ Nếu có 2 người nói thật thì 2 trong 4 số b, c, d, ebằng 2 và hai số còn lại khác 2.

- Nếu 2 số còn lại lớn hơn 2 thì ta có thể xét bộ (0, 2, 2, d, e) với d, e > 2, có tất cả C 2+3−1 2 = 6 bộ.

- Nếu trong hai số còn lại, có 1 số bằng 1 thì ta chỉ cần xét bộ

(0, 2, 2, 1, 1) vì các bộ còn lại đã được đếm ở trên.

Suy ra có 6 + 1 = 7 bộ thỏa mãn.

+ Nếu có 3 người nói thật thì 3 trong 4 số b, c, d, e bằng 3 và số còn lại khác 3.

- Nếu số còn lại lớn hơn 3 thì ta xét bộ (0, 3, 3, 3, e), có 2 bộ thỏa mãn.

- Nếu số còn lại bé hơn 3 thì ta có 2 bộ

(0, 3, 3, 3, 1) và (0, 3, 3, 3, 2), vì bộ (0, 3, 3, 3, 1) đã được đếm ở trên nên chỉ có 1 bộ thỏa mãn.

Do đó ta được 2 + 1 = 3 bộ.

+ Nếu bốn người nói thật thì b= c = d = e= 4 có 1 trường hợp.

Do đó có tất cả 20 + 7 + 3 + 1 = 31 bộ.

Vậy có tất cả 126 + 0 + 1 + 5 + 14 + 31 = 177 bộ.

Luận văn đã trình bày được những vấn đề sau đây

1 Sưu tầm và phận dạng các bài toán Số học có liên quan đến yếu tố

Tổ hợp và ngược lại.

2 Trình bày được ứng dụng của bất biến và đơn biến trong giải toán

3 Áp dụng phương pháp phân hoạch tập hợp giải một số dạng toán

4 Vận dụng được kiến thức cơ bản về số học và tổ hợp trong chương trình Toán sơ cấp để giải các bài toán thi chọn học sinh giỏi.

Luận văn cần bổ sung một số vấn đề sau

Để nâng cao chất lượng luận văn, cần bổ sung thêm các dạng bài toán Số học - Tổ hợp khác, bao gồm Lí thuyết chia hết với Tổ hợp và Lí thuyết trò chơi với Tổ hợp.

2 Luận văn có thể phát triển theo hướng Toán ứng dụng.

Ngày đăng: 11/08/2021, 08:59

TÀI LIỆU CÙNG NGƯỜI DÙNG

TÀI LIỆU LIÊN QUAN

w